Putnam All Questions

download Putnam All Questions

of 109

Transcript of Putnam All Questions

  • 7/26/2019 Putnam All Questions

    1/109

    The Forty-Sixth Annual William Lowell Putnam Competition

    Saturday, December 7, 1985

    A1 Determine, with proof, the number of ordered triples

    of sets which have the property that

    (i)

    ,and

    (ii)

    .

    Express your answer in the form

    , where

    are nonnegative integers.

    A2 Let be an acute triangle. Inscribe a rectangle in

    with one side along a side of . Then inscribe a rectan-

    gle in the triangle formed by the side of

    opposite

    the side on the boundary of , and the other two sides of

    , with one side along the side of

    . For any polygon

    , let

    denote the area of

    . Find the maximumvalue, or show that no maximum exists, of

    ,

    where ranges over all triangles and

    over all rect-

    angles as above.

    A3 Let

    be a real number. For each integer

    , define

    a sequence

    ,

    by the condition

    Evaluate

    .

    A4 Define a sequence

    by

    and

    for

    . Which integers between 00 and 99 inclusive oc-

    cur as the last two digits in the decimal expansion of

    infinitely many ?

    A5 Let

    . For

    which integers ,

    is

    ?

    A6 If

    is a polynomial with

    real coefficients , then set

    Let

    . Find, with proof, a polyno-

    mial

    with real coefficients such that

    (i)

    , and

    (ii)

    for every integer

    .

    B1 Let be the smallest positive integer for which there ex-ist distinct integers

    such that the

    polynomial

    has exactly nonzero coefficients. Find, with proof, a

    set of integers

    for which this min-

    imum is achieved.

    B2 Define polynomials

    for

    by

    ,

    for

    , and

    for

    . Find, with proof, the explicit factorization of

    into powers of distinct primes.

    B3 Let

    ......

    .... . .

    be a doubly infinite array of positive integers, and sup-

    pose each positive integer appears exactly eight times

    in the array. Prove that

    for some pair ofpositive integers

    .

    B4 Let be the unit circle

    . A point is chosen

    randomly on the circumference and another point

    is chosen randomly from the interior of (these points

    are chosen independently and uniformly over their do-

    mains). Let be the rectangle with sides parallel to the

    and

    -axes with diagonal . What is the probability

    that no point of lies outside of ?

    B5 Evaluate

    . You may assume

    that

    .

    B6 Let be a finite set of real

    matrices

    ,

    , which form a group under matrix multi-

    plication. Suppose that

    , where

    denotes the trace of the matrix

    . Prove that

    is the

    zero matrix.

  • 7/26/2019 Putnam All Questions

    2/109

    The Forty-Seventh Annual William Lowell Putnam Competition

    Saturday, December 6, 1986

    A1 Find, with explanation, the maximum value off(x) =x3 3xon the set of all real numbersxsatisfyingx4 +36

    13x2.

    A2 What is the units (i.e., rightmost) digit of

    1020000

    10100 + 3

    ?

    A3 Evaluate

    n=0Arccot(n2+n+1), where Arccot tfor

    t 0denotes the number in the interval0< /2withcot = t.

    A4 A transversalof annnmatrixAconsists ofnentriesofA, no two in the same row or column. Let f(n) bethe number ofnn matricesAsatisfying the followingtwo conditions:

    (a) Each entryi,j ofAis in the set {1, 0, 1}.(b) The sum of the n entries of a transversal is the

    same for all transversals ofA.

    An example of such a matrixA is

    A=

    1 0 10 1 0

    0 1 0

    .

    Determine with proof a formula forf(n)of the form

    f(n) =a1bn1 +a2bn2 +a3bn3 +a4,

    where theais andbis are rational numbers.

    A5 Suppose f1(x), f2(x), . . . , f n(x)are functions ofnrealvariablesx = (x1, . . . , xn) with continuous second-order partial derivatives everywhere on Rn. Suppose

    further that there are constantscij such that

    fixj

    fjxi

    =cij

    for alli and j ,1 i n,1 j n. Prove that thereis a function g(x)on Rn such thatfi + g/xiis linearfor all i, 1 i n. (A linear function is one of theform

    a0+a1x1+a2x2+ +anxn.)

    A6 Let a1, a2, . . . , an be real numbers, and letb1, b2, . . . , bn be distinct positive integers. Sup-pose that there is a polynomial f(x) satisfying theidentity

    (1 x)nf(x) = 1 +n

    i=1

    aixbi .

    Find a simple expression (not involving any sums) fof(1) in terms ofb1, b2, . . . , bn andn (but independenofa1, a2, . . . , an).

    B1 Inscribe a rectangle of base b and height h in a circlof radius one, and inscribe an isosceles triangle in th

    region of the circle cut off by one base of the rectangl

    (with that side as the base of the triangle). For whavalue ofh do the rectangle and triangle have the samarea?

    B2 Prove that there are only a finite number of possibilitie

    for the ordered tripleT = (x y, y z, z x), wherx,y,z are complex numbers satisfying the simultaneous equations

    x(x 1) + 2yz = y(y 1) + 2zx = z(z 1) + 2xy,and list all such triplesT.

    B3 Let consist of all polynomials inx with integer coefficients. Forf andg in and m a positive integer, lef g (mod m) mean that every coefficient offis an integral multiple ofm. Let n and p be positivintegers withp prime. Given thatf , g , h , rand s are i withrf+sg 1 (mod p) and f g h (mod p)prove that there exist F and G in with F f(modp),G g (mod p), andF G h (mod pn).

    B4 For a positive real numberr, letG(r)be the minimumvalue of|r m2 + 2n2| for all integers m and nProve or disprove the assertion that limrG(r) exists and equals 0.

    B5 Let f(x,y,z) = x2 + y2 + z2 + xyz . Lep(x,y,z), q(x,y,z), r(x,y,z) be polynomials witreal coefficients satisfying

    f(p(x,y,z), q(x,y,z), r(x,y,z)) =f(x,y,z).

    Prove or disprove the assertion that the sequencep,q,consists of some permutation ofx,y,z, where thnumber of minus signs is 0 or 2.

    B6 SupposeA,B, C,D are n nmatrices with entries ia fieldF, satisfying the conditions thatABT andCDT

    are symmetric and ADT BCT = I. HereI is thnnidentity matrix, and ifMis annnmatrix,MTis its transpose. Prove thatATD CTB= I.

  • 7/26/2019 Putnam All Questions

    3/109

    The Forty-Eighth Annual William Lowell Putnam Competition

    Saturday, December 5, 1987

    A1 Curves

    and are defined in the plane as fol-

    lows:

    Prove that

    .

    A2 The sequence of digits

    is obtained by writing the positive integers in order. If

    the

    -th digit in this sequence occurs in the part ofthe sequence in which the -digit numbers are placed,

    define

    to be . For example,

    because

    the 100th digit enters the sequence in the placement of

    the two-digit integer 55. Find, with proof,

    .

    A3 For all real

    , the real-valued function

    satis-

    fies

    (a) If

    for all real

    , must

    for all

    real

    ? Explain.

    (b) If

    for all real

    , must

    for all

    real

    ? Explain.A4 Let

    be a polynomial, with real coefficients, in three

    variables and be a function of two variables such that

    for all real

    and such that

    ,

    , and

    . Also let

    be complex numbers

    with

    and

    . Find

    .

    A5 Let

    Prove or disprove that there is a vector-valued function

    with the following properties:

    (i)

    have continuous partial derivatives for

    all

    ;

    (ii)

    for all

    ;

    (iii)

    .

    A6 For each positive integer

    , let

    be the number of

    zeroes in the base 3 representation of

    . For which pos-

    itive real numbers

    does the series

    converge?

    B1 Evaluate

    B2 Let

    and be integers with

    ,

    and

    . Prove that

    B3 Let be a field in which

    . Show that the set

    of solutions to the equation

    with

    and

    in is given by

    and

    where runs through the elements of such that

    .

    B4 Let

    and let

    and

    for

    . For each of

    and

    ,

    prove that the limit exists and find it or prove that thelimit does not exist.

    B5 Let be the

    -dimensional vector

    . Let

    be a

    matrix of complex numbers such that

    whenever

    , with complex

    ,

    not all zero, then at least one of the

    is not real. Prove

    that for arbitrary real numbers

    , there are

    complex numbers

    such that

    ...

    ...

    (Note: if

    is a matrix of complex numbers,

    isthe matrix whose entries are the real parts of the entries

    of

    .)

    B6 Let be the field of

    elements, where is an odd

    prime. Suppose is a set of

    distinct nonzero

    elements of with the property that for each

    in

    , exactly one of and

    is in . Let

    be the num-

    ber of elements in the intersection

    .

    Prove that

    is even.

  • 7/26/2019 Putnam All Questions

    4/109

    The Forty-Ninth Annual William Lowell Putnam Competition

    Saturday, December 3, 1988

    A1 Let

    be the region consisting of the points of thecartesian plane satisfying both

    and

    .

    Sketch the region

    and find its area.

    A2 A not uncommon calculus mistake is to believe that the

    product rule for derivatives says that . If

    , determine, with proof, whether there exists

    an open interval

    and a nonzero function defined

    on

    such that this wrong product rule is true for

    in

    .

    A3 Determine, with proof, the set of real numbers for

    which

    converges.

    A4 (a) If every point of the plane is painted one of three

    colors, do there necessarily exist two points of the

    same color exactly one inch apart?

    (b) What if three is replaced by nine?

    A5 Prove that there exists auniquefunction from the set

    of positive real numbers to

    such that

    and

    for all

    .

    A6 If a linear transformation on an -dimensional vector

    space has

    eigenvectors such that any of them

    are linearly independent, does it follow that is a scalar

    multiple of the identity? Prove your answer.

    B1 A composite (positive integer) is a product with

    and not necessarily distinct integers in

    .

    Show that every composite is expressible as

    , with

    positive integers.

    B2 Prove or disprove: If and are real numbers with

    and

    , then

    .

    B3 For every in the set

    of positive inte-

    gers, let

    be the minimum value of

    for all

    nonnegative integers and

    with

    . Find, with

    proof, the smallest positive real number with

    for all

    .

    B4 Prove that if

    is a convergent series of positive

    real numbers, then so is

    .

    B5 For positive integers , let be the by

    skew-symmetric matrix for which each entry in the first subdiagonals below the main diagonal is 1 and each

    of the remaining entries below the main diagonal is -1.

    Find, with proof, the rank of

    . (According to one

    definition, the rank of a matrix is the largest such that

    there is a

    submatrix with nonzero determinant.)

    One may note that

    B6 Prove that there exist an infinite number of ordered pairs

    of integers such that for every positive integer

    ,

    the number

    is a triangular number if and only if

    is a triangular number. (The triangular numbers are the

    with in

    .)

  • 7/26/2019 Putnam All Questions

    5/109

    The Fiftieth Annual William Lowell Putnam Competition

    Saturday, December 2, 1989

    A1 How many primes among the positive integers, writtenas usual in base 10, are alternating 1s and 0s, begin-

    ning and ending with 1?

    A2 Evaluate

    where and

    are positive.

    A3 Prove that if

    then

    (Here

    is a complex number and

    .)

    A4 If is an irrational number,

    , is there afinite game with an honest coin such that the probabil-

    ity of one player winning the game is ? (An honest

    coin is one for which the probability of heads and the

    probability of tails are both

    . A game is finite if with

    probability 1 it must end in a finite number of moves.)

    A5 Let be a positive integer and let

    be a regular

    -gon inscribed in the unit circle. Show that

    there is a positive constant , independent of

    , with

    the following property. For any points inside

    there

    are two distinct vertices

    and

    of such that

    Here

    denotes the distance between the points

    and

    .

    A6 Let

    be a formal power series

    with coefficients in the field of two elements. Let

    if every block of zeros in the binary

    expansion of has an even number

    of zeros in the block

    otherwise.

    (For example,

    because

    and

    because

    ) Prove that

    B1 A dart, thrown at random, hits a square target. Assum-

    ing that any two parts of the target of equal area are

    equally likely to be hit, find the probability that the point

    hit is nearer to the center than to any edge. Express your

    answer in the form

    , where

    are inte-

    gers.

    B2 Let be a non-empty set with an associative opera-tion that is left and right cancellative (

    implies

    , and

    implies

    ). Assume that for

    every in

    the set

    is finite.

    Must be a group?

    B3 Let be a function on

    , differentiable and satis-

    fying

    for

    . Assume that

    for

    (so

    that

    tends rapidly to

    as

    increases). For a

    non-negative integer, define

    (sometimes called the th moment of

    ).

    a) Express in terms of .

    b) Prove that the sequence

    always con-

    verges, and that the limit is

    only if

    .

    B4 Can a countably infinite set have an uncountable collec-

    tion of non-empty subsets such that the intersection of

    any two of them is finite?

    B5 Label the vertices of a trapezoid (quadrilateral

    with two parallel sides) inscribed in the unit circle

    as

    so that

    is parallel to

    and

    are in counterclockwise order. Let

    ,

    and

    denote the lengths of the line segments

    ,

    and , where E is the point of intersection of the di-

    agonals of , and is the center of the circle. Deter-

    mine the least upper bound of

    over all such for

    which

    , and describe all cases, if any, in which it

    is attained.

    B6 Let

    be a point chosen at random from

    the -dimensional region defined by

    Let be a continuous function on

    with

    . Set

    and

    . Show that

    the expected value of the Riemann sum

    is

    , where

    is a polynomial of degree ,

    independent of , with

    for

    .

  • 7/26/2019 Putnam All Questions

    6/109

    The 51st William Lowell Putnam Mathematical Competition

    Saturday, December 8, 1990

    A1 Let

    T0= 2, T1= 3, T2= 6,

    and forn3,Tn= (n + 4)Tn14nTn2+ (4n 8)Tn3.

    The first few terms are

    2, 3, 6, 14, 40, 152, 784, 5168, 40576.

    Find, with proof, a formula for Tn of the form Tn =An+ Bn, where{An} and{Bn} are well-known se-quences.

    A2 Is

    2 the limit of a sequence of numbers of the form3

    n 3m(n, m= 0, 1, 2, . . . )?A3 Prove that any convex pentagon whose vertices (no

    three of which are collinear) have integer coordinates

    must have area greater than or equal to 5/2.

    A4 Consider a paper punch that can be centered at any pointof the plane and that, when operated, removes from the

    plane precisely those points whose distance from the

    center is irrational. How many punches are needed to

    remove every point?

    A5 IfA and B are square matrices of the same size suchthat ABAB = 0, does it follow that BABA = 0?

    A6 If X is a finite set, let X denote the number of ele-ments inX. Call an ordered pair(S, T) of subsets of{1, 2, . . . , n} admissibleifs >|T| for eachsS, andt >|S|for eachtT. How many admissible orderedpairs of subsets of{1, 2, . . . , 10} are there? Prove youranswer.

    B1 Find all real-valued continuously differentiable func-tionsfon the real line such that for allx,

    (f(x))2 = x

    0

    [(f(t))2 + (f(t))2] dt + 1990.

    B2 Prove that for |x|< 1, |z|> 1,

    1 +

    j=1

    (1 + xj)Pj = 0,

    wherePj is

    (1 z)(1 zx)(1 zx2) (1 zxj1)(z x)(z x2)(z x3) (z xj) .

    B3 Let S be a set of 2 2 integer matrices whose entries aij (1) are all squares of integers and, (2) satisfy aij 200. Show that ifShas more than 50387(= 154 152 15 + 2) elements, then it has two elements that commute.

    B4 LetGbe a finite group of orderngenerated byaandbProve or disprove: there is a sequence

    g1, g2, g3, . . . , g2n

    such that

    (1) every element ofG occurs exactly twice, and

    (2) gi+1equalsgiaorgibfori= 1, 2, . . . , 2n. (Interpretg2n+1as g1.)

    B5 Is there an infinite sequencea0, a1, a2, . . . of nonzerreal numbers such that for

    n = 1, 2, 3, . . . the polyno

    mial

    pn(x) = a0+ a1x + a2x2 + + anxn

    has exactlyn distinct real roots?

    B6 LetSbe a nonempty closed bounded convex set in thplane. LetK be a line and t a positive number. LeL1 and L2 be support lines for Sparallel to K1, anletL be the line parallel toKand midway betweenLandL2. LetBS(K, t)be the band of points whose distance fromLis at most(t/2)w, wherew is the distanc

    betweenL1and L2. What is the smallestt such that

    SK

    BS(K, t)=

    for allS? (Kruns over all lines in the plane.)

  • 7/26/2019 Putnam All Questions

    7/109

    The Fifty-Second William Lowell Putnam Mathematical Competition

    Saturday, December 7, 1991

    A1 A

    rectangle has vertices as

    and

    . It rotates

    clockwise about the point

    . It

    then rotates

    clockwise about the point

    , then

    clockwise about the point

    , and finally,

    clockwise about the point

    . (The side originally

    on the -axis is now back on the

    -axis.) Find the

    area of the region above the -axis and below the curve

    traced out by the point whose initial position is (1,1).

    A2 Let and

    be different

    matrices with real en-

    tries. If and , can be

    invertible?

    A3 Find all real polynomials

    of degree

    for

    which there exist real numbers

    such that

    1.

    and

    2.

    where

    denotes the derivative of

    .

    A4 Does there exist an infinite sequence of closed

    discs

    in the plane, with centers

    , respectively, such that

    1. the have no limit point in the finite plane,

    2. the sum of the areas of the

    is finite, and3. every line in the plane intersects at least one of the

    ?

    A5 Find the maximum value of

    for .

    A6 Let denote the number of sums of positive inte-

    gers

    which add up to with

    Let

    denote the number of

    which

    add up to , with

    1.

    2. each is in the sequence

    de-fined by

    ,

    , and

    and

    3. if

    then every element in

    appears at least once as a

    .

    Prove that

    for each

    .

    (For example, because the relevant sums are

    and

    because

    the relevant sums are

    )

    B1 For each integer

    , let

    , where

    is the greatest integer with

    . Define a sequence

    by

    and

    for

    .

    For what positive integers is this sequence eventually

    constant?

    B2 Suppose and

    are non-constant, differentiable, real-

    valued functions defined on

    . Furthermore,

    suppose that for each pair of real numbers and

    ,

    If

    , prove that

    for all

    .

    B3 Does there exist a real number such that, if and

    are integers greater than , then an

    rectangle

    may be expressed as a union of

    and

    rect-

    angles, any two of which intersect at most along their

    boundaries?

    B4 Suppose is an odd prime. Prove that

    B5 Let be an odd prime and let

    denote (the field of)

    integers modulo . How many elements are in the set

    B6 Let

    and be positive numbers. Find the largest num-

    ber , in terms of

    and , such that

    for all

    with

    and for all ,

    .

    (Note:

    .)

  • 7/26/2019 Putnam All Questions

    8/109

    The 53rd William Lowell Putnam Mathematical Competition

    Saturday, December 5, 1992

    A-1 Prove thatf(n) = 1nis the only integer-valued func-tion defined on the integers that satisfies the following

    conditions.

    (i) f(f(n)) = n, for all integersn;

    (ii) f(f(n+ 2) + 2) = n for all integersn;

    (iii) f(0) = 1.

    A-2 DefineC()to be the coefficient ofx1992 in the powerseries aboutx = 0of(1 +x). Evaluate

    10

    C(y 1)

    1992k=1

    1

    y+k

    dy.

    A-3 For a given positive integerm, find all triples (n,x,y)of positive integers, withnrelatively prime tom, whichsatisfy

    (x2 +y2)m = (xy)n.

    A-4 Letfbe an infinitely differentiable real-valued functiondefined on the real numbers. If

    f

    1

    n

    =

    n2

    n2 + 1, n= 1, 2, 3, . . . ,

    compute the values of the derivatives f(k)(0), k =1, 2, 3, . . . .

    A-5 For each positive integern, letan= 0(or 1) if the num-ber of 1s in the binary representation ofn is even (orodd), respectively. Show that there do not exist positiveintegersk and msuch that

    ak+j =ak+m+j =ak+2m+j ,

    for0 j m 1.

    A-6 Four points are chosen at random on the surface of a

    sphere. What is the probability that the center of the

    sphere lies inside the tetrahedron whose vertices are atthe four points? (It is understood that each point is in-

    dependently chosen relative to a uniform distribution on

    the sphere.)

    B-1 Let S be a set ofn distinct real numbers. LetAS bethe set of numbers that occur as averages of two distinct

    elements ofS. For a givenn 2, what is the smallestpossible number of elements inAS?

    B-2 For nonnegative integers n and k , defineQ(n, k)to bethe coefficient ofxk in the expansion of(1 +x+x2 +x3)n. Prove that

    Q(n, k) =k

    j=0

    n

    j

    n

    k 2j

    ,

    whereab

    is the standard binomial coefficient. (Re

    minder: For integersaandbwitha 0,

    ab

    = a!

    b!(ab)

    for0 b a, with ab= 0otherwise.)

    B-3 For any pair (x, y) of real numbers, a sequenc(an(x, y))n0 is defined as follows:

    a0(x, y) = x,

    an+1(x, y) = (an(x, y))

    2 +y2

    2 , forn 0.

    Find the area of the region

    {(x, y)|(an(x, y))n0 converges}.

    B-4 Letp(x) be a nonzero polynomial of degree less tha1992 having no nonconstant factor in common with

    x3 x. Let

    d1992

    dx1992

    p(x)

    x3 x

    =

    f(x)

    g(x)

    for polynomialsf(x) and g(x). Find the smallest possible degree off(x).

    B-5 LetDndenote the value of the(n 1) (n 1)determinant

    3 1 1 1 11 4 1 1 11 1 5 1 11 1 1 6 1...

    ......

    .... . .

    ...

    1 1 1 1 n+ 1

    .

    Is the setDnn!

    n2

    bounded?

    B-6 Let M be a set of realn nmatrices such that

    (i) I M, whereIis then nidentity matrix;

    (ii) ifA Mand B M, then either AB MoAB M, but not both;

    (iii) ifA M andB M, then eitherAB = BAoAB= BA;

    (iv) ifA M andA=I, there is at least oneB Msuch thatAB = BA.

    Prove that M contains at mostn2 matrices.

  • 7/26/2019 Putnam All Questions

    9/109

    The 54th William Lowell Putnam Mathematical Competition

    Saturday, December 4, 1993

    A1 The horizontal liney = c intersects the curvey = 2x3x3 in the first quadrant as in the figure. Findc so thatthe areas of the two shaded regions are equal. [Figure

    not included. The first region is bounded by they-axis,the line y = c and the curve; the other lies under thecurve and above the liney = cbetween their two pointsof intersection.]

    A2 Let (xn)n0 be a sequence of nonzero real numberssuch that x2n xn1xn+1 = 1 for n = 1, 2, 3, . . . .Prove there exists a real number a such that xn+1 =axn xn1 for alln 1.

    A3 Let Pn be the set of subsets of {1, 2, . . . , n}. Letc(n, m) be the number of functions f : Pn {1, 2, . . . , m} such that f(AB) = min{f(A), f(B)}.Prove that

    c(n, m) =mj=1

    jn.

    A4 Let x1, x2, . . . , x19be positive integers each of which isless than or equal to 93. Lety1, y2, . . . , y93 be positiveintegers each of which is less than or equal to 19. Prove

    that there exists a (nonempty) sum of somexis equalto a sum of someyjs.

    A5 Show that

    10100

    x2 xx3 3x+ 1

    2dx+

    111

    1

    101

    x2 x

    x3 3x+ 1

    2dx+

    1110

    101

    100

    x2 x

    x3 3x+ 1

    2dx

    is a rational number.

    A6 The infinite sequence of 2s and 3s

    2, 3, 3, 2, 3, 3, 3, 2, 3, 3, 3, 2, 3, 3, 2, 3, 3,

    3, 2, 3, 3, 3, 2, 3, 3, 3, 2, 3, 3, 2, 3, 3, 3, 2, . . .

    has the property that, if one forms a second sequencethat records the number of 3s between successive 2s,

    the result is identical to the given sequence. Show that

    there exists a real numberrsuch that, for anyn, thenthterm of the sequence is 2 if and only ifn = 1 + rmfor some nonnegative integer m. (Note:x denotes thelargest integer less than or equal tox.)

    B1 Find the smallest positive integern such that for everintegermwith0< m < 1993, there exists an integerkfor which

    m

    1993 N.B4 Forn 1, letdn be the greatest common divisor of th

    entries ofAn I, where

    A=

    3 24 3

    and I=

    1 00 1

    .

    Show thatlimn dn= .

    B5 For any real number , define the function f(x) =x. Letnbe a positive integer. Show that there existan such that for1 k n,

    fk(n2) =n2 k= fk(n2).

    B6 For any integern, set

    na= 101a 100 2a.

    Show that for0 a,b,c,d 99,na+ nb nc+ n(mod 10100)implies {a, b} = {c, d}.

  • 7/26/2019 Putnam All Questions

    11/109

    The Fifty-Sixth William Lowell Putnam Mathematical Competition

    Saturday, December 2, 1995

    A1 Let

    be a set of real numbers which is closed under

    multiplication (that is, if and

    are in

    , then so is ).

    Let and be disjoint subsets of

    whose union is

    . Given that the product of any three(not necessarilydistinct) elements of is in and that the product of

    any three elements of is in

    , show that at least one

    of the two subsets

    is closed under multiplication.

    A2 For what pairs of positive real numbers does the

    improper integral

    converge?

    A3 The number has nine (not necessarily dis-

    tinct) decimal digits. The number

    is such

    that each of the nine 9-digit numbers formed by replac-ing just one of the digits

    is

    by the corre-

    sponding digit (

    ) is divisible by 7. The

    number

    is related to

    is the same

    way: that is, each of the nine numbers formed by replac-

    ing one of the

    by the corresponding

    is divisible by

    7. Show that, for each ,

    is divisible by 7. [For

    example, if

    , then may be 2

    or 9, since

    and

    are multiples of

    7.]

    A4 Suppose we have a necklace of beads. Each bead is

    labeled with an integer and the sum of all these labels

    is

    . Prove that we can cut the necklace to form a

    string whose consecutive labels

    satisfy

    for

    A5 Let

    be differentiable (real-valued) func-

    tions of a single variable which satisfy

    ..

    .

    ..

    .

    for some constants

    . Suppose that for all ,

    as

    . Are the functions

    necessarily linearly dependent?

    A6 Suppose that each of people writes down the numbers

    1,2,3 in random order in one column of a

    matrix,

    with all orders equally likely and with the orders for

    different columns independent of each other. Let the

    row sums

    of the resulting matrix be rearranged

    (if necessary) so that . Show that for some

    , it is at least four times as likely that both

    and

    as that .

    B1 For a partition of

    , let

    be

    the number of elements in the part containing

    . Prove

    that for any two partitions and

    , there are two dis-

    tinct numbers

    and in such

    that

    and

    . [A partition of

    a set

    is a collection of disjoint subsets (parts) whose

    union is

    .]

    B2 An ellipse, whose semi-axes have lengths and , rolls

    without slipping on the curve . How are

    related, given that the ellipse completes one rev-

    olution when it traverses one period of the curve?

    B3 To each positive integer with

    decimal digits, we as-

    sociate the determinant of the matrix obtained by writ-

    ing the digits in order across the rows. For example, for

    , to the integer 8617 we associate

    . Find, as a function of , the sum of all the determi-

    nants associated with

    -digit integers. (Leading digits

    are assumed to be nonzero; for example, for ,

    there are 9000 determinants.)

    B4 Evaluate

    Express your answer in the form

    , where

    are integers.

    B5 A game starts with four heaps of beans, containing 3,4,5

    and 6 beans. The two players move alternately. A move

    consists of takingeither

    a) one bean from a heap, provided at least two beans

    are left behind in that heap, or

    b) a complete heap of two or three beans.

    The player who takes the last heap wins. To win thegame, do you want to move first or second? Give a

    winning strategy.

    B6 For a positive real number , define

    Prove that cannot be expressed as the dis-

    joint union of three sets

    and

    . [As

    usual,

    is the greatest integer

    .]

  • 7/26/2019 Putnam All Questions

    12/109

    Solutions to the Fifty-Sixth William Lowell Putnam Mathematical Competition

    Saturday, December 2, 1995

    Kiran Kedlaya

    A1 Suppose on the contrary that there exist

    with

    and

    with

    . Then

    while

    , contradiction.

    A2 The integral converges iff . The easiest proof

    uses big-O notation and the fact that

    for

    . (Here

    means

    bounded by a constant times

    .)

    So

    hence

    and similarly

    Hence the integral were looking at is

    The term

    is bounded by a constant times

    , whose integral converges. Thus we only have todecide whether

    converges. But

    has divergent integral, so we get convergence if and

    only if (in which case the integral telescopes any-

    way).

    A3 Let and

    be the numbers

    and

    , re-

    spectively. We are given that

    and

    for

    . Sum the first relation over

    and we

    get

    , or

    .

    Now add the first and second relations for any particu-

    lar value of and we get

    . But we know

    is divisible by 7, and 10

    is coprime to 7, so

    .

    A4 Let

    , so that

    . These form a cyclic sequence that doesnt change

    when you rotate the necklace, except that the entire se-

    quence gets translated by a constant. In particular, it

    makes sense to choose

    for which is maximum and

    make that one

    ; this way

    for all , which gives

    , but the right side may be

    replaced by

    since the left side is an integer.

    A5 Everyone (presumably) knows that the set of solutions

    of a system of linear first-order differential equations

    with constant coefficients is

    -dimensional, with ba-

    sis vectors of the form

    (i.e. a function times

    a constant vector), where the

    are linearly indepen-

    dent. In particular, our solution

    can be written as

    .

    Choose a vector

    orthogonal to

    but not to

    . Since

    as

    , the same is true of

    ;

    but that is simply

    . In other words, if

    , then

    must also go to 0.

    However, it is easy to exhibit a solution which does

    not go to 0. The sum of the eigenvalues of the matrix

    , also known as the trace of

    , being the sum

    of the diagonal entries of

    , is nonnegative, so

    has

    an eigenvalue with nonnegative real part, and a cor-

    responding eigenvector

    . Then

    is a solution that

    does not go to 0. (If is not real, add this solution to

    its complex conjugate to get a real solution, which still

    doesnt go to 0.)

    Hence one of the

    , say

    , is zero, in which case

    for all

    .

    A6 View this as a random walk/Markov process with states

    the triples of integers with sum 0, correspond-

    ing to the difference between the first, second and third

    rows with their average (twice the number of columns).

    Adding a new column adds on a random permutation

    of the vector

    . I prefer to identify the triple

    with the point

    in

    the plane, where

    is a cube root of unity. Then adding

    a new column corresponds to moving to one of the six

    neighbors of the current position in a triangular lattice.

    What wed like to argue is that for large enough

    , the

    ratio of the probabilities of being in any two particular

    states goes to 1. Then in fact, well see that eventually,about six times as many matrices have

    than

    . This is a pain to prove, though,

    and in fact is way more than we actually need.

    Let and

    be the probability that we are at the

    origin, or at a particular point adjacent to the origin,

    respectively. Then

    . (In fact,

    is

    times the sum of the probabilities of being at each

    neighbor of the origin at time

    , but these are all

    .)

  • 7/26/2019 Putnam All Questions

    13/109

    So the desired result, which is that

    for

    some large

    , is equivalent to

    .

    Suppose on the contrary that this is not the case; then

    for some constant

    . However, if

    , the probability that we chose each of the six

    types of moves

    times is already

    ,

    which by Stirlings approximation is asymptotic to a

    constant times

    . This term alone is bigger than

    , so we must have

    for some

    . (In fact, we must have

    for any

    .)

    B1 For a given , no more than three different values of

    are possible (four would require one part each of

    size at least 1,2,3,4, and thats already more than 9 el-

    ements). If no such

    exist, each pair

    occurs for at most 1 element of

    , and since there are

    only

    possible pairs, each must occur exactly once.

    In particular, each value of

    must occur 3 times.

    However, clearly any given value of

    occurs

    times, where is the number of distinct partitions of

    that size. Thus

    can occur 3 times only if it equals

    1 or 3, but we have three distinct values for which it

    occurs, contradiction.

    B2 For those who havent taken enough physics, rolling

    without slipping means that the perimeter of the ellipse

    and the curve pass at the same rate, so all were saying

    is that the perimeter of the ellipse equals the length of

    one period of the sine curve. So set up the integrals:

    Let

    in the second integral and write 1 as

    and you get

    Since the left side is increasing as a function of , we

    have equality if and only if

    .

    B3 For

    we obviously get 45, while for

    the

    answer is 0 because it both changes sign (because de-terminants are alternating) and remains unchanged (by

    symmetry) when you switch any two rows other than

    the first one. So only

    is left. By the multilin-

    earity of the determinant, the answer is the determinant

    of the matrix whose first (resp. second) row is the sum

    of all possible first (resp. second) rows. There are 90

    first rows whose sum is the vector

    , and 100

    second rows whose sum is

    . Thus the answer

    is

    B4 The infinite continued fraction is defined as the limit

    of the sequence

    .

    Notice that the sequence is strictly decreasing (by in-

    duction) and thus indeed has a limit , which satisfies

    , or rewriting,

    .

    Moreover, we want the greater of the two roots.

    Now how to compute the eighth root of ? Notice that

    if

    satisfies the quadratic

    , then we

    have

    Clearly, then, the positive square roots of the quadratic

    satisfy the quadratic

    . Thus we compute that

    is the greater

    root of

    ,

    is the greater root

    of

    , and

    is the greater root of

    , otherwise known as

    .

    B5 This problem is dumb if you know the Sprague-

    Grundy theory of normal impartial games (see Conway,Berlekamp and Guy,Winning Ways, for details). Ill de-

    scribe how it applies here. To each position you assign

    a nim-value as follows. A position with no moves (in

    which case the person to move has just lost) takes value

    0. Any other position is assigned the smallest number

    not assigned to a valid move from that position.

    For a single pile, one sees that an empty pile has value

    0, a pile of 2 has value 1, a pile of 3 has value 2, a pile

    of 4 has value 0, a pile of 5 has value 1, and a pile of 6

    has value 0.

    You add piles just like in standard Nim: the nim-value

    of the composite of two games (where at every turn you

    pick a game and make a move there) is the base 2 ad-dition without carries (i.e. exclusive OR) of the nim-

    values of the constituents. So our starting position, with

    piles of 3, 4, 5, 6, has nim-value

    .

    A position is a win for the player to move if and only if

    it has a nonzero value, in which case the winning strat-

    egy is to always move to a 0 position. (This is always

    possible from a nonzero position and never from a zero

    position, which is precisely the condition that defines

    the set of winning positions.) In this case, the winning

    move is to reduce the pile of 3 down to 2, and you can

    easily describe the entire strategy if you so desire.

    B6 Obviously

    have to be greater than 1, and no twocan both be rational, so without loss of generality as-

    sume that and

    are irrational. Let

    denote the fractional part of

    . Then

    if and

    only if

    . In particular, this

    means that

    contains

    elements, and similarly. Hence for every integer

    ,

    2

  • 7/26/2019 Putnam All Questions

    14/109

    Dividing through by

    and taking the limit as

    shows that

    . That in turn implies

    that for all

    ,

    Our desired contradiction is equivalent to showing that

    the left side actually takes the value 1 for some

    .

    Since the left side is an integer, it suffices to show that

    for some

    .

    A result in ergodic theory (the two-dimensional version

    of the Weil equidistribution theorem) states that if

    are linearly independent over the rationals, then the set

    of points

    is dense (and in fact equidis-

    tributed) in the unit square. In particular, our claim def-

    initely holds unless

    for some integers

    .

    On the other hand, suppose that such a relation

    does hold. Since and

    are irrational, by the

    one-dimensional Weil theorem, the set of points

    is dense in the set of

    in the

    unit square such that

    is an integer. It is simpleenough to show that this set meets the region

    unless

    is an integer, and that

    would imply that

    , a quantity between 0 and

    1, is an integer. We have our desired contradiction.

    3

  • 7/26/2019 Putnam All Questions

    15/109

    The Fifty-Seventh William Lowell Putnam Mathematical Competition

    Saturday, December 7, 1996

    A1 Find the least number

    such that for any two squares of

    combined area 1, a rectangle of area

    exists such that

    the two squares can be packed in the rectangle (without

    interior overlap). You may assume that the sides of thesquares are parallel to the sides of the rectangle.

    A2 Let

    and

    be circles whose centers are 10 units

    apart, and whose radii are 1 and 3. Find, with proof, the

    locus of all points

    for which there exists points on

    and

    on

    such that

    is the midpoint of the line

    segment

    .

    A3 Suppose that each of 20 students has made a choice of

    anywhere from 0 to 6 courses from a total of 6 courses

    offered. Prove or disprove: there are 5 students and 2

    courses such that all 5 have chosen both courses or all 5

    have chosen neither course.

    A4 Let be the set of ordered triples

    of distinct

    elements of a finite set

    . Suppose that

    1.

    if and only if

    ;

    2.

    if and only if

    ;

    3.

    and

    are both in if and only if

    and

    are both in

    .

    Prove that there exists a one-to-one function from

    to

    such that implies .

    Note:

    is the set of real numbers.

    A5 If is a prime number greater than 3 and

    ,

    prove that the sum

    of binomial coefficients is divisible by

    .

    A6 Let

    be a constant. Give a complete descrip-

    tion, with proof, of the set of all continuous functions

    such that

    for all

    .

    Note that

    denotes the set of real numbers.

    B1 Define a selfishset to be a set which has its own cardi-

    nality (number of elements) as an element. Find, with

    proof, the number of subsets of

    which are

    minimalselfish sets, that is, selfish sets none of whose

    proper subsets is selfish.

    B2 Show that for every positive integer

    ,

    B3 Given that

    , find,

    with proof, the largest possible value, as a function of

    (with ), of

    B4 For any square matrix

    , we can define

    by the

    usual power series:

    Prove or disprove: there exists a

    matrix

    with

    real entries such that

    B5 Given a finite string of symbols and , we write

    for the number of s in minus the number

    of s. For example,

    . We

    call a string balanced if every substring of (con-

    secutive symbols of) has

    . Thus,

    is not balanced, since it contains the sub-

    string

    . Find, with proof, the number of bal-

    anced strings of length .

    B6 Let

    be the vertices of a

    convex polygon which contains the origin in its inte-

    rior. Prove that there exist positive real numbers and

    such that

  • 7/26/2019 Putnam All Questions

    16/109

    Solutions to the Fifty-Eighth William Lowell Putnam Mathematical Competition

    Saturday, December 7, 1996

    Manjul Bhargava and Kiran Kedlaya

    A-1 If and are the sides of two squares with combined

    area 1, then

    . Suppose without loss of gen-

    erality that

    . Then the shorter side of a rectangle

    containing both squares without overlap must be at least , and the longer side must be at least

    . Hence the

    desired value of is the maximum of

    .

    To find this maximum, we let

    with . Then we are to maximize

    with equality for . Hence this value is the de-

    sired value of .

    A-2 Let

    and

    be the centers of

    and

    , respec-

    tively. (We are assuming

    has radius 1 and

    has

    radius 3.) Then the desired locus is an annulus centered

    at the midpoint of

    , with inner radius 1 and outer

    radius 2.

    For a fixed point on

    , the locus of the midpoints of

    the segments for lying on is the image of

    under a homothety centered at of radius

    , which

    is a circle of radius

    . As varies, the center of this

    smaller circle traces out a circle

    of radius

    (again

    by homothety). By considering the two positions of

    on the line of centers of the circles, one sees that

    is

    centered at the midpoint of

    , and the locus is now

    clearly the specified annulus.

    A-3 The claim is false. There are

    ways to choose

    3 of the 6 courses; have each student choose a different

    set of 3 courses. Then each pair of courses is chosen by

    4 students (corresponding to the four ways to complete

    this pair to a set of 3 courses) and is not chosen by 4

    students (corresponding to the 3-element subsets of the

    remaining 4 courses).

    Note: Assuming that no two students choose the same

    courses, the above counterexample is unique (up to per-

    muting students). This may be seen as follows: Given a

    group of students, suppose that for any pair of courses

    (among the six) there are at most 4 students taking both,

    and at most 4 taking neither. Then there are at most

    pairs

    , where is a student, and

    is a set of two courses of which is taking either both

    or none. On the other hand, if a student is taking

    courses, then he/she occurs in

    such

    pairs

    . As

    is minimized for

    , it follows

    that every student occurs in at least

    such

    pairs

    . Hence there can be at most

    stu-

    dents, with equality only if each student takes 3 courses,

    and for each set of two courses, there are exactly 4 stu-

    dents who take both and exactly 4 who take neither.

    Since there are only 4 ways to complete a given pair

    of courses to a set of 3, and only 4 ways to choose 3

    courses not containing the given pair, the only way for

    there to be 20 students (under our hypotheses) is if allsets of 3 courses are in fact taken. This is the desired

    conclusion.

    However, Robin Chapman has pointed out that the so-

    lution is not unique in the problem as stated, because a

    given selection of courses may be made by more than

    one student. One alternate solution is to identify the 6

    courses with pairs of antipodal vertices of an icosahe-

    dron, and have each student pick a different face and

    choose the three vertices touching that face. In this ex-

    ample, each of 10 selections is made by a pair of stu-

    dents.

    A-4 In fact, we will show that such a function

    exists withthe property that

    if and only if

    for some cyclic permutation

    of

    . We proceed by induction on the number of el-

    ements in . If and , then

    choose with , otherwise choose

    with .

    Now let be an element of

    and

    .

    Let be the elements of labeled such that

    . We claim that there ex-

    ists a unique such that ,

    where hereafter

    .

    We show existence first. Suppose no such exists; then

    for all

    , we have

    .

    This holds by property 1 for and by induction on

    in general, noting that

    Applying this when

    , we get

    ,

  • 7/26/2019 Putnam All Questions

    17/109

    contradicting the fact that

    . Hence ex-

    istence follows.

    Now we show uniqueness. Suppose

    ; then for any , we have

    by the assumption on

    . Therefore

    so

    . The case

    is ruled out by

    and the case is similar.

    Finally, we put in if , and

    otherwise; an analysis similar to that

    above shows that has the desired property.

    A-5 (due to Lenny Ng) For

    , divides

    and

    where the congruence

    means that

    is a rational number whose numerator, in reduced form,

    is divisible by . Hence it suffices to show that

    We distinguish two cases based on . First

    suppose

    , so that

    . Then

    since

    .

    Now suppose

    , so that

    . A similar

    argument gives

    A-6 We first consider the case

    ; we shall show in

    this case must be constant. The relation

    proves that is an even function. Let

    be the

    roots of , both of which are real. If

    ,

    define

    and

    for each positiveinteger . By induction on ,

    for all , so the sequence tends to a limit

    which is a

    root of not less than

    . Of course this means

    . Since for all and

    ,

    we conclude

    , so is constant on

    .

    If

    and is defined as before, then by in-

    duction,

    . Note that the sequence can

    be defined because

    ; the latter follows by noting

    that the polynomial is positive at and

    has its minimum at

    , so both roots are greater

    than . In any case, we deduce that is also constant

    on

    .

    Finally, suppose

    . Now define

    . Given that

    , we have

    . Thus

    if we had

    for all , by the same argument as in

    the first case we deduce

    and so

    .

    Actually, this doesnt happen; eventually we have

    , in which case by what we

    have already shown. We conclude that is a constant

    function. (Thanks to Marshall Buck for catching an in-

    accuracy in a previous version of this solution.)

    Now suppose

    . Then the sequence defined

    by and

    is strictly increasing

    and has no limit point. Thus if we define on

    as any continuous function with equal values on the

    endpoints, and extend the definition from

    to

    by the relation

    , and

    extend the definition further to

    by the relation

    , the resulting function has the desired

    property. Moreover, any function with that property

    clearly has this form.

    B-1 Let denote the set

    , and let denote

    the number of minimal selfish subsets of . Then the

    number of minimal selfish subsets of not containing

    2

  • 7/26/2019 Putnam All Questions

    18/109

    is equal to

    . On the other hand, for any mini-

    mal selfish subset of containing , by subtracting 1

    from each element, and then taking away the element from the set, we obtain a minimal selfish subset

    of

    (since and cannot both occur in a selfish

    set). Conversely, any minimal selfish subset of

    gives rise to a minimal selfish subset of containing by the inverse procedure. Hence the number of min-

    imal selfish subsets of

    containing

    is

    . Thuswe obtain

    . Since

    , we

    have , where denotes the th term of the

    Fibonacci sequence.

    B-2 By estimating the area under the graph of using up-

    per and lower rectangles of width 2, we get

    Since

    , we have, upon expo-

    nentiating and taking square roots,

    using the fact that

    .

    B-3 View

    as an arrangement of the numbers

    on a circle. We prove that the optimal ar-

    rangement is

    To show this, note that if is a pair of adjacent num-

    bers and is another pair (read in the same order

    around the circle) with and , then the seg-

    ment from to can be reversed, increasing the sum

    by

    Now relabel the numbers so they appear in order as fol-lows:

    where without loss of generality we assume

    . By considering the pairs

    and

    and using the trivial fact

    , we

    deduce

    . We then compare the pairs

    and

    , and using that

    , we deduce

    . Continuing in this

    fashion, we prove that

    and

    so

    for

    , i.e. that the optimal ar-

    rangement is as claimed. In particular, the maximum

    value of the sum is

    Alternate solution: We prove by induction that the value

    given above is an upper bound; it is clearly a lower

    bound because of the arrangement given above. As-

    sume this is the case for . The optimal arrangement

    for is obtained from some arrangement for by

    inserting between some pair of adjacent terms.

    This operation increases the sum by

    , which is an increasing function of

    both and . In particular, this difference is maximal

    when and equal and

    . Fortunately, this

    yields precisely the difference between the claimed up-

    per bound for and the assumed upper bound for ,

    completing the induction.

    B-4 Suppose such a matrix exists. If the eigenvalues of

    (over the complex numbers) are distinct, then there

    exists a complex matrix such that

    is

    diagonal. Consequently,

    is diagonal. But then

    must be diagonalizable, a con-

    tradiction. Hence the eigenvalues of are the same,

    and has a conjugate

    over the complex

    numbers of the form

    A direct computation shows that

    Since

    and

    are conjugate, their eigenvalues

    must be the same, and so we must have . This

    implies , so that is the identity matrix,

    as must be , a contradiction. Thus cannot exist.

    Alternate solution (due to Craig Helfgott and AlexPopa): Define both

    and

    by the usual power

    series. Since commutes with itself, the power series

    identity

    holds. But if is the given matrix, then by the above

    identity,

    must equal

    which is

    3

  • 7/26/2019 Putnam All Questions

    19/109

    a nilpotent matrix. Thus is also nilpotent. How-

    ever, the square of any

    nilpotent matrix must be

    zero (e.g., by the Cayley-Hamilton theorem). This is a

    contradiction.

    B-5 Consider a

    checkerboard, in which we write an -letter string, one letter per square. If the string is

    balanced, we can cover each pair of adjacent squares

    containing the same letter with a

    domino, and

    these will not overlap (because no three in a row can

    be the same). Moreover, any domino is separated from

    the next by an even number of squares, since they must

    cover opposite letters, and the sequence must alternate

    in between.

    Conversely, any arrangement of dominoes where ad-

    jacent dominoes are separated by an even number of

    squares corresponds to a unique balanced string, once

    we choose whether the string starts with or . In

    other words, the number of balanced strings is twice

    the number of acceptable domino arrangements.

    We count these arrangements by numbering the squares and distinguishing whether the dominoes

    start on even or odd numbers. Once this is decided, one

    simply chooses whether or not to put a domino in each

    eligible position. Thus we have

    arrangements in

    the first case and

    in the second, but note that

    the case of no dominoes has been counted twice. Hence

    the number of balanced strings is

    B-6 We will prove the claim assuming only that the convex

    hull of the points contains the origin in its in-

    terior. (Thanks to Marshall Buck for pointing out that

    the last three words are necessary in the previous sen-

    tence!) Let

    so that the left-hand

    side of the given equation is

    (1)

    Now note that (1) is the gradient of the function

    and so it suffices to show has a critical point. We will

    in fact show has a global minimum.

    Clearly we have

    Note that this maximum is positive for :

    if we had for all , then the subset

    of the -plane would be a half-plane

    containing all of the points , whose convex hull

    would then not contain the origin, a contradiction.

    The function

    is clearly continuous on

    the unit circle

    , which is compact. Hence it

    has a global minimum , and so for all ,

    In particular,

    on the disk of radius

    . Since

    , the infimum of

    is the same over the entire -plane as over this disk,

    which again is compact. Hence attains its infimal

    value at some point in the disk, which is the desired

    global minimum.

    Noam Elkies has suggested an alternate solution as fol-

    lows: for , draw the loop traced by (1) as

    travels counterclockwise around the circle .

    For , this of course has winding number 0 about

    any point, but for large, one can show this loop has

    winding number 1 about the origin, so somewhere in

    between the loop must pass through the origin. (Prov-

    ing this latter fact is a little tricky.)

    4

  • 7/26/2019 Putnam All Questions

    20/109

    The Fifty-Eighth William Lowell Putnam Mathematical Competition

    Saturday, December 6, 1997

    A1 A rectangle,

    , has sides

    and

    . A triangle

    has

    as the intersection of the al-

    titudes,

    the center of the circumscribed circle,

    the

    midpoint of

    , and

    the foot of the altitude from .

    What is the length of

    ?

    A2 Players

    are seated around a table, and

    each has a single penny. Player 1 passes a penny to

    player 2, who then passes two pennies to player 3.

    Player 3 then passes one penny to Player 4, who passes

    two pennies to Player 5, and so on, players alternately

    passing one penny or two to the next player who still

    has some pennies. A player who runs out of pennies

    drops out of the game and leaves the table. Find an in-

    finite set of numbers

    for which some player ends upwith all

    pennies.

    A3 Evaluate

    A4 Let be a group with identity and a

    function such that

    whenever

    . Prove that thereexists an element such that

    is a homomorphism (i.e.

    for all

    ).

    A5 Let denote the number of ordered

    -tuples of posi-

    tive integers

    such that

    . Determine whether

    is even or odd.

    A6 For a positive integer

    and any real number , define

    recursively by

    ,

    , and for

    ,

    Fix

    and then take to be the largest value for which

    . Find

    in terms of

    and ,

    .

    B1 Let

    denote the distance between the real number

    and the nearest integer. For each positive integer

    ,

    evaluate

    (Here

    denotes the minimum of and

    .)

    B2 Let be a twice-differentiable real-valued function sat-

    isfying

    where

    for all real

    . Prove that

    is

    bounded.

    B3 For each positive integer

    , write the sum

    in the form

    , where

    and

    are relatively prime

    positive integers. Determine all

    such that 5 does not

    divide

    .

    B4 Let

    denote the coefficient of

    in the expansion

    of

    . Prove that for all [integers]

    ,

    B5 Prove that for

    ,

    terms

    terms

    B6 The dissection of the 345 triangle shown below (into

    four congruent right triangles similar to the original) has

    diameter

    . Find the least diameter of a dissection of

    this triangle into four parts. (The diameter of a dissec-

    tion is the least upper bound of the distances betweenpairs of points belonging to the same part.)

  • 7/26/2019 Putnam All Questions

    21/109

    Solutions to the Fifty-Eighth William Lowell Putnam Mathematical Competition

    Saturday, December 6, 1997

    Manjul Bhargava, Kiran Kedlaya, and Lenny Ng

    A1 The centroid

    of the triangle is collinear with and

    (Euler line), and the centroid lies two-thirds of the

    way from to

    . Therefore

    is also two-thirds of

    the way from to

    , so

    . Since the triangles

    and

    are similar (theyre right triangles and

    ), we have

    , or

    . Now

    , but

    , so

    A2 We show more precisely that the game terminates with

    one player holding all of the pennies if and only if

    or

    for some

    . First sup-

    pose we are in the following situation for some

    .

    (Note: for us, a move consists of two turns, starting

    with a one-penny pass.)

    Except for the player to move, each player has

    pennies;

    The player to move has at least pennies.

    We claim then that the game terminates if and only if

    the number of players is a power of 2. First supposethe number of players is even; then after complete

    rounds, every other player, starting with the player who

    moved first, will have more pennies than initially, and

    the others will all have 0. Thus we are reduced to the

    situation with half as many players; by this process, we

    eventually reduce to the case where the number of play-

    ers is odd. However, if there is more than one player,

    after two complete rounds everyone has as many pen-

    nies as they did before (here we need

    ), so the

    game fails to terminate. This verifies the claim.

    Returning to the original game, note that after one com-

    plete round,

    players remain, each with 2 pennies

    except for the player to move, who has either 3 or 4 pen-nies. Thus by the above argument, the game terminates

    if and only if

    is a power of 2, that is, if and only

    if

    or

    for some .

    A3 Note that the series on the left is simply

    .

    By integration by parts,

    and so by induction,

    Thus the desired integral is simply

    A4 In order to have for all , we must in par-

    ticular have this for , and so we take

    .

    We first note that

    and so

    commutes with

    for all

    . Next, we

    note that

    and using the commutativity of , we deduce

    or

    , as desired.

    A5 We may discard any solutions for which

    , since

    those come in pairs; so assume

    . Similarly, wemay assume that

    ,

    ,

    ,

    .

    Thus we get the equation

    Again, we may assume

    and

    , so we get

    ; and

    , so

    . This implies that

    , which by

    counting has 5 solutions. Thus

    is odd.

    A6 Clearly

    is a polynomial in of degree , so it suf-

    fices to identify values of for which

    . We

    claim these are

    for

    ; in

    this case,

    is the coefficient of

    in the polynomial

    . This can be verified bynoticing that

    satisfies the differential equation

    (by logarithmic differentiation) or equivalently,

  • 7/26/2019 Putnam All Questions

    22/109

    and then taking the coefficient of

    on both sides:

    In particular, the largest such is , and

    for

    .

    Greg Kuperberg has suggested an alternate approach to

    show directly that

    is the largest root, withoutcomputing the others. Note that the condition

    states that

    is an eigenvector of the matrix

    otherwise

    with eigenvalue . By the Perron-Frobenius theorem,

    has a unique eigenvector with positive entries, whose

    eigenvalue has modulus greater than or equal to that of

    any other eigenvalue, which proves the claim.

    B1 It is trivial to check that

    for

    , that

    for

    , that

    for

    , and that

    for

    . Therefore the desired sum is

    B2 It suffices to show that

    is bounded for ,

    since

    satisfies the same equation as

    . But

    then

    so that

    for

    .

    B3 The only such are the numbers 14, 2024, 100104,

    and 120124. For the proof let

    and introduce the auxiliary function

    It is immediate (e.g., by induction) that

    (mod ) for

    (mod 5) re-

    spectively, and moreover, we have the equality

    where denotes the largest integer such that

    . We wish to determine those such that the

    above sum has nonnegative 5valuation. (By the 5

    valuation of a number we mean the largest integer

    such that

    is an integer.)

    If

    , then the last term in the above sum

    has 5valuation , since

    ,

    ,

    each have valu-

    ation 0; on the other hand, all other terms must have

    5valuation strictly larger than . It follows that

    has 5valuation exactly ; in particular,

    has non-

    negative 5valuation in this case if and only if ,

    i.e., , 2, or 3.

    Suppose now that

    . Then we must also

    have

    . The former condition im-

    plies that the last term of the above sum is

    , which has 5valuation

    .

    It is clear that

    (mod 25); hence if

    equals 20 or 24, then the secondtolast term

    of the above sum (if it exists) has valuation at least

    . The thirdtolast term (if it exists) is of

    the form

    , so that the sum of the last term andthe third to last term takes the form

    .

    Since

    can be congruent only to 0,1, or -1 (mod 5),

    and

    (mod 5), we conclude that the sum of the

    last term and thirdtolast term has valuation

    ,

    while all other terms have valuation strictly higher.

    Hence

    has nonnegative5valuation in this case only

    when

    , leading to the values

    (arising from

    ), 20,24 (arising from

    and

    and 24 resp.), 101, 102, 103, and 104 (arising from

    ,

    ) and 120, 121, 122, 123, and

    124 (arising from ,

    ).

    Finally, suppose

    and

    , 22,

    or 23. Then as before, the first condition implies thatthe last term of the sum in (*) has valuation

    ,

    while the second condition implies that the secondto

    last term in the same sum has valuation

    . Hence

    all terms in the sum (*) have 5valuation strictly higher

    than

    , except for the secondtolast term, and

    therefore

    has 5valuation

    in this case. In

    particular,

    is integral (mod 5) in this case if and only

    if

    , which gives the additional values

    , 22,

    and 23.

    B4 Let

    be the given sum (note that

    is nonzero precisely for

    . Since

    we have

    2

  • 7/26/2019 Putnam All Questions

    23/109

    By computing

    , we may eas-

    ily verify by induction that

    and

    for all

    . (Alternate so-

    lution suggested by John Rickert: write

    , and note note that is the

    coefficient of

    in

    .)

    B5 Define the sequence

    ,

    for .

    It suffices to show that for every

    ,

    for some

    . We do this by induction on , with

    being obvious.

    Write

    , where is odd. It suffices to show

    that

    modulo

    and modulo , for some

    . For the former, we only need

    ,

    but clearly

    by induction on . For the lat-

    ter, note that

    as long as

    , where is the Eu-

    ler totient function. By hypothesis, this occurs for some

    . (Thanks to Anoop Kulkarni for

    catching a lethal typo in an earlier version.)

    B6 The answer is

    . Place the triangle on the carte-

    sian plane so that its vertices are at

    . It is easy to check that the five points

    and

    are

    all in the triangle and have distance at least

    apart

    from each other (note that

    ); thus any dis-

    section of the triangle into four parts must have diame-

    ter at least

    .

    We now exhibit a dissection with least diameter

    .(Some variations of this dissection are possible.) Put

    ,

    ,

    ,

    , and divide

    into

    the convex polygonal regions

    ,

    ,

    ,

    ; each region has diameter

    , as can be

    verified by checking the distance between each pair of

    vertices of each polygon. (One need only check for the

    pentagon: note that and

    are contained in

    circular sectors centered at and

    , respectively, of ra-

    dius

    and angle less than

    , and that

    is

    a rectangle with diagonal

    .)

    3

  • 7/26/2019 Putnam All Questions

    24/109

    The 59th William Lowell Putnam Mathematical Competition

    Saturday, December 5, 1998

    A1 A right circular cone has base of radius 1 and height 3.A cube is inscribed in the cone so that one face of the

    cube is contained in the base of the cone. What is the

    side-length of the cube?

    A2 Lets be any arc of the unit circle lying entirely in thefirst quadrant. LetA be the area of the region lyingbelows and above thex-axis and let B be the area ofthe region lying to the right of they-axis and to the leftofs. Prove thatA +B depends only on the arc length,and not on the position, ofs.

    A3 Let fbe a real function on the real line with continuousthird derivative. Prove that there exists a pointa suchthat

    f(a)

    f(a)

    f(a)

    f(a)

    0.

    A4 Let A1 = 0 and A2 = 1. For n > 2, the num-berAn is defined by concatenating the decimal expan-sions ofAn1 and An2 from left to right. For ex-ample A3 = A2A1 = 10, A4 = A3A2 = 101,A5 = A4A3 = 10110, and so forth. Determine allnsuch that11 divides An.

    A5 LetFbe a finite collection of open discs in R2 whoseunion contains a set E R2. Show that there is apairwise disjoint subcollection D1, . . . , Dn inF suchthat

    E nj=13Dj.Here, ifD is the disc of radius rand centerP, then3Dis the disc of radius3rand centerP.

    A6 LetA, B,Cdenote distinct points with integer coordi-nates in R2. Prove that if

    (|AB| + |BC|)2 0.

    B2 Given a point(a, b)with0< b < a, determine the minimum perimeter of a triangle with one vertex at (a, b)one on thex-axis, and one on the liney =x. You maassume that a triangle of minimum perimeter exists.

    B3 let H be the unit hemisphere{(x,y,z) : x2 +y2 +z2 = 1, z 0}, C the unit circle{(x,y, 0) : x2 +y2 = 1}, and Pthe regular pentagon inscribed in CDetermine the surface area of that portion ofH lyin

    over the planar region insideP, and write your answein the form A sin + B cos , where A,B,,are reanumbers.

    B4 Find necessary and sufficient conditions on positive in

    tegersmandnso that

    mn1

    i=0

    (1)i/m+i/n = 0.

    B5 LetN

    be the positive integer with 1998 decimal digits

    all of them 1; that is,

    N= 1111 11.

    Find the thousandthdigit after the decimal point of

    N

    B6 Prove that, for any integersa, b, c, there exists a positive integern such that

    n3 + an2 + bn + cis not a

    integer.

  • 7/26/2019 Putnam All Questions

    25/109

    Solutions to the 59th William Lowell Putnam Mathematical Competition

    Saturday, December 5, 1998

    Manjul Bhargava, Kiran Kedlaya, and Lenny Ng

    A1 Consider the plane containing both the axis of the cone

    and two opposite vertices of the cubes bottom face.

    The cross section of the cone and the cube in this plane

    consists of a rectangle of sides and inscribed in

    an isosceles triangle of base and height , where is

    the side-length of the cube. (The side of the rect-

    angle lies on the base of the triangle.) Similar triangles

    yield

    , or

    A2 First solution: to fix notation, let be the area of re-

    gion

    , and be the area of

    ; further let

    denote the area of sector

    , which only depends

    on the arc length of

    . If

    denotes the area of tri-angle

    , then we have

    and

    . But clearly

    and

    , and so

    .

    O

    D

    E

    F G

    H

    I

    Second solution: We may parametrize a point in by

    any of ,

    , or

    . Then and

    are

    just the integrals of

    and

    over the appropriate

    intervals; thus

    is the integral of

    (mi-

    nus because the limits of integration are reversed). But

    , and so

    is precisely the

    radian measure of . (Of course, one can perfectly well

    do this problem by computing the two integrals sepa-

    rately. But whats the fun in that?)

    A-3 If at least one of

    ,

    ,

    , or

    van-

    ishes at some point

    , then we are done. Hence wemay assume each of , , , and

    is either strictly positive or strictly negative on the

    real line. By replacing by if necessary,

    we may assume ; by replacing by

    if necessary, we may assume . (No-

    tice that these substitutions do not change the sign of

    .) Now

    implies that

    is increasing, and

    implies that

    is convex, so that

    for all

    and

    . By letting

    increase in the latter inequality,

    we see that

    must be positive for sufficiently

    large ; it follows that

    for all . Similarly,

    and

    imply that

    for all

    . Therefore

    for all , and

    we are done.

    A-4 The number of digits in the decimal expansion of

    is the Fibonacci number , where

    ,

    ,

    and

    for

    . It follows that

    the sequence

    , modulo 11, satisfies the recursion

    . (Notice that the recur-

    sion for

    depends only on the value of

    modulo2.) Using these recursions, we find that and

    modulo 11, and that

    and

    mod-

    ulo 2. It follows that

    (mod 11) for all

    . We find that among

    , and

    , only

    vanishes modulo 11. Thus 11 divides

    if and only if

    for some nonnegative integer

    .

    A5 Define the sequence by the following greedy algo-

    rithm: let

    be the disc of largest radius (breaking ties

    arbitrarily), let

    be the disc of largest radius not meet-

    ing

    , let

    be the disc of largest radius not meeting

    or

    , and so on, up to some final disc

    . To see

    that

    , consider a point in ; if it lies inone of the

    , we are done. Otherwise, it lies in a disc

    of radius

    , which meets one of the

    having radius

    (this is the only reason a disc can be skipped

    in our algorithm). Thus the centers lie at a distance

    , and so every point at distance less than

    from the center of lies at distance at most

    from the center of the corresponding

    .

    A6 Recall the inequalities

    (AM-GM) and

    (Law of Sines).

    Also recall that the area of a triangle with integer

    coordinates is half an integer (if its vertices lie at

    , the area is

    ), and that

    if and have integer coordinates, then

    is an

    integer (Pythagoras). Now observe that

    and that the first and second expressions are both inte-

    gers. We conclude that

  • 7/26/2019 Putnam All Questions

    26/109

    , and so

    ; that is, is a right angle and

    ,

    as desired.

    B1 Notice that

    (difference of squares). The latter is easily seen (e.g.,

    by AM-GM) to have minimum value 6 (achieved at

    ).

    B2 Consider a triangle as described by the problem; la-

    bel its vertices

    so that

    , lies on

    the -axis, and

    lies on the line

    . Further

    let

    be the reflection of in the

    -axis,

    and let

    be the reflection of in the line

    . Then and

    , and so

    the perimeter of

    is

    . It is clear that

    this lower bound can be achieved; just set (resp.

    )

    to be the intersection between the segment and the

    -axis (resp. line

    ); thus the minimum perimeter

    is in fact

    .

    B3 We use the well-known result that the surface area of

    the sphere cap

    is simply

    . (This result is easily verified using

    calculus; we omit the derivation here.) Now the desired

    surface area is just minus the surface areas of five

    identical halves of sphere caps; these caps, up to isome-

    try, correspond to

    being the distance from the center

    of the pentagon to any of its sides, i.e.,

    .

    Thus the desired area is

    (i.e.,

    ).

    B4 For convenience, define

    , so that

    the given sum is

    . If

    and are both odd, then

    is the sum of an

    odd number of s, and thus cannot be zero. Now

    consider the case where and have opposite par-

    ity. Note that

    for all

    integers

    . Thus

    and

    ; this implies that

    is odd, and so

    for all . It follows

    that

    if and

    have opposite parity.

    Now suppose that

    and

    are both even.Then

    for all , so

    can be computed

    as twice the sum over only even indices:

    Thus

    vanishes if and only if

    vanishes

    (if

    , then and have opposite parity

    and so

    also vanishes).

    Piecing our various cases together, we easily deduce

    that

    if and only if the highest powers of 2

    dividing and

    are different.

    B5 Write

    . Then

    where

    . Now the digits after the deci-

    mal point of

    are given by

    , while the

    digits after the decimal point of

    are given by

    . It follows that the first 1000

    digits of are given by

    ; in particu-

    lar, the thousandth digit is .

    B6 First solution: Write

    . Note

    that

    and

    have the same parity, and recall

    that any perfect square is congruent to 0 or 1 (mod 4).

    Thus if and

    are perfect squares, they are

    congruent mod 4. But

    (mod4), which is not divisible by 4 if

    and

    have opposite

    parity.

    Second solution: We prove more generally that for any

    polynomial

    with integer coefficients which is not

    a perfect square, there exists a positive integer such

    that

    is not a perfect square. Of course it suffices

    to assume

    has no repeated factors, which is to say

    and its derivative

    are relatively prime.

    In particular, if we carry out the Euclidean algorithm

    on

    and

    without dividing, we get an in-

    teger (the discriminant of

    ) such that the great-

    est common divisor of

    and

    divides

    for any . Now there exist infinitely many primes

    such that divides for some : if there were

    only finitely many, say,

    , then for any di-

    visible by

    , we have

    , that is,

    is not divisible

    by

    , so must be

    , but then

    takes some

    value infinitely many times, contradiction. In particu-

    lar, we can choose some such not dividing

    , and

    choose such that

    divides

    . Then

    (write out the Taylor series

    of the left side); in particular, since does not divide

    , we can find some such that

    is di-

    visible by but not by

    , and so is not a perfect square.

    Third solution: (from David Rusin, David Savitt, and

    Richard Stanley independently) Assume that

    is a square for all

    . For sufficiently large

    ,

    2

  • 7/26/2019 Putnam All Questions

    27/109

    thus if is a large even perfect square, we have

    . We conclude this is an

    equality of polynomials, but the right-hand side is not a

    perfect square for an even non-square, contradiction.

    (The reader might try generalizing this approach to ar-

    bitrary polynomials. A related argument, due to Greg

    Kuperberg: write

    as

    times a

    power series in and take two finite differences to

    get an expression which tends to 0 as , contra-

    diction.)

    Note: in case

    has no repeated fac-

    tors, it is a square for only finitely many , by a theorem

    of Siegel; work of Baker gives an explicit (but large)

    bound on such . (I dont know whether the graders

    will accept this as a solution, though.)

    3

  • 7/26/2019 Putnam All Questions

    28/109

    The 60th William Lowell Putnam Mathematical Competition

    Saturday, December 4, 1999

    A-1 Find polynomials

    ,

    , and

    , if they exist,such that for all

    ,

    if

    if

    if

    .

    A-2 Let

    be a polynomial that is nonnegative for all

    real

    . Prove that for some , there are polynomials

    ) such that

    A-3 Consider the power series expansion

    Prove that, for each integer

    , there is an integer

    such that

    A-4 Sum the series

    A-5 Prove that there is a constant such that, if

    is a

    polynomial of degree 1999, then

    A-6 The sequence

    is defined by

    and, for

    ,

    Show that, for all n,

    is an integer multiple of .

    B-1 Right triangle has right angle at and

    ; the point is chosen on

    so that

    ; the point is chosen on

    so that

    .

    The perpendicular to

    at meets

    at

    . Evalu-

    ate

    .

    B-2 Let

    be a polynomial of degree such that

    , where

    is a quadratic polynomial and

    is the second derivative of

    . Show that if

    has at least two distinct roots then it must have

    distinct roots.

    B-3 Let

    . For

    , let

    where the sum ranges over all pairs

    of positive

    integers satisfying the indicated inequalities. Evaluate

    B-4 Let

    be a real function with a continuous third deriva-

    tive such that

    are positive forall

    . Suppose that

    for all

    . Show that

    for all

    .

    B-5 For an integer

    , let

    . Evaluate the

    determinant of the

    matrix

    , where

    is

    the

    identity matrix and

    has entries

    for all

    .

    B-6 Let

    be a finite set of integers, each greater than 1.

    Suppose that for each integer there is some

    such that

    or

    . Show that

    there exist

    such that

    is prime.

  • 7/26/2019 Putnam All Questions

    29/109

    Solutions to the 60th William Lowell Putnam Mathematical Competition

    Saturday, December 4, 1999

    Manjul Bhargava, Kiran Kedlaya, and Lenny Ng

    A1 Note that ifr(x)and s(x)are any two functions, then

    max(r, s) = (r+s+ |r s|)/2.

    Therefore, ifF(x)is the given function, we have

    F(x) = max{3x 3, 0} max{5x, 0} + 3x+ 2= (3x 3 + |3x+ 3|)/2

    (5x+ |5x|)/2 + 3x+ 2= |(3x+ 3)/2| |5x/2| x+1

    2,

    so we may setf(x) = (3x+ 3)/2,g(x) = 5x/2, andh(x) = x+ 12 .

    A2 First solution: First factorp(x) = q(x)r(x), whereqhas all real roots andr has all complex roots. Noticethat each root ofq has even multiplicity, otherwise pwould have a sign change at that root. Thusq(x)has asquare roots(x).

    Now write r(x) =k

    j=1(x aj)(x aj) (possiblebecauser has roots in complex conjugate pairs). Writek

    j=1(x aj) = t(x) + iu(x) with t, x having realcoefficients. Then forxreal,

    p(x) = q(x)r(x)

    =s(x)

    2

    (t(x) +iu(x))(t(x) +iu(x))= (s(x)t(x))2 + (s(x)u(x))2.

    (Alternatively, one can factor r(x) as a product ofquadratic polynomials with real coefficients, write each

    as a sum of squares, then multiply together to get a sum

    of many squares.)

    Second solution: We proceed by induction on the de-

    gree ofp, with base case where p has degree 0. As inthe first solution, we may reduce to a smaller degree

    in case p has any real roots, so assume it has none.Then p(x) > 0 for all real x, and since p(x) forx

    ,phas a minimum valuec. Nowp(x)

    c

    has real roots, so as above, we deduce that p(x) cisa sum of squares. Now add one more square, namely

    (

    c)2, to getp(x)as a sum of squares.

    A3 First solution: Computing the coefficient ofxn+1 in theidentity(12x x2)m=0amxm = 1 yields therecurrencean+1 = 2an + an1; the sequence{an}is then characterized by this recurrence and the initial

    conditionsa0 = 1, a1= 2.

    Define the sequence {bn} by b2n = a2n1 +

    a2n, b2n+1 = an(an1+an+1).Then

    2b2n+1+b2n= 2anan+1+ 2an1an+a2n1+a

    2n

    = 2anan+1+an1an+1+a2n

    =a2n+1+a2n = b2n+2,

    and similarly 2b2n + b2n1 = b2n+1, so that{bn}satisfies the same recurrence as{an}. Since furtherb0 = 1, b1 = 2 (where we use the recurrence for {an}to calcul